You are on page 1of 77

1. Under which of the following circumstances may ‘capital gains’ arise?

1 When there is an increase in the sales of a product.


2 When there is a natural increase in the value of the property owned.

3 When you purchase a painting and there is a growth in its value due to increase in its popularity.
Which of the above the statements is/ are correct?

a) 1 only
b) 2 and 3 only
c) 2 only
d) 1, 2 and 3

Answer: B

Explanation: An increase in the value of a capital asset (investment or real estate) that gives it
a higher worth than the purchase price. The gain is not realized until the asset is sold. A capital
gain may be short term (one year or less) or long term (more than one year) and must be
claimed on income taxes.

2. Consider the following :

1. Income tax.
2. Interest Tax.
3. Securities Transaction Tax .
Which of the above is/are Direct Tax/Taxes ?

a) 1 only
b) 1 and 3 only
c) 2 and 3 only
d) 1, 2 and 3
Answer: D

Explanation :Direct tax is a tax or a kind of charge that is imposed directly to taxpayer and
cannot be shifted on to others.

● Corporation tax,
● Income tax,
● Securities transaction tax,
● Banking Cash Transaction tax,
● Fringe Benefit tax,
● Wealth tax,
● Interest tax,
● Gift tax
3. Consider the following regarding Voluntary Compliance Encouragement Scheme: Change this

1. Any person who has tax dues to declare can make a declaration in terms of the provisions of
VCES.
2. In VCES beside interest and penalty, immunity would also be available from any other
proceeding under the Finance Act, 1994.
Which of the above the statements is/ are correct?

a) 1 only
b) 2 only
c) Both 1 and 2
d) None of the above

Answer : C

Explanation: Any person who has tax dues to declare can make a declaration in terms of the
provisions of VCES. If such person does not already have a service tax registration he will be
required to take registration before making such declaration.

The Service Tax VCES Scheme was a Voluntary Compliance Encouragement Scheme
(VCES)that made it easier for businesses and individuals to declare their past due taxes.

4.3. Consider the following statements :

1. A regressive tax is defined as a tax whose rate increases as the payer's income
increases.
2. A progressive tax,on the other hand,is one whose rate increases as the payer's income
decreases.
Which of the above the statements is/ are correct?

a) 1 only
b) 2 only
c) Both 1 and 2
d) None of the above
Answer: D

Explanation: A progressive tax is defined as a tax whose rate increases as the payer's
income increases.A regressive tax,on the other hand,is one whose rate increases as the payer's
income decreases.Fore.g.consumption taxes are called as regressive, as rich spend less
(and hence pay less taxes) on consumption goods as a proportion of their income.
5.4. An economy with high tax buoyancy is most likely to have:

1. Greater tax collections with higher economic growth

2. Lower contribution of tax receipts to revenue receipts.

Which of the above the statements is/ are correct?

a) 1 only
b) 2 only
c) Both 1 and 2
d) None of the above
Answer: A

Explanation: Tax buoyancy refers to the response of tax collections with higher economic
growth.If tax buoyancy is good, and suppose economy grows rapidly, the tax collections
of government will improve, for e.g. due to higher corporate tax.Buoyancy does not
calculate the share of government receipts, i.e. the part of the non-tax revenue is not dealt
by the concept of buoyancy.

6.5. Consider the following regarding Tax Expenditure:

1. It relates to the expenditures incurred by the Government in the collection of taxes.

2. It is the revenue forgone by the government on account of exemptions on income and


corporate tax.

Which of the above statements is/are right?

a) 1 only

b) 2 only

c) Both 1 and 2

d) None of the above

Answer: B

Explanation:Tax Expenditures, as the word might indicate, does not relate to the expenditures
incurred by the Government in the collection of taxes. Rather it refers to the opportunity cost of
taxing at concessional rates, or the opportunity cost of giving exemptions, deductions, rebates,
deferrals credits etc. to the tax payers. Tax expenditures indicate how much more revenue
could have been collected by the Government if not for such measures. In other words, it shows
the extent of indirect subsidy enjoyed by the tax payers in the country.
7.6. Consider the following about Cess:

1. Cess is levied only for particular purpose and cannot be used for any other purpose.

2. collections from a cess are required to be kept outside of the Consolidated Fund of India to
be spent only on the specific purpose for which it was levied.

Which of the above statements is/are right?

a) 1 only

b) 2 only

c) Both 1 and 2

d) None of the above

Answer : C

Explanation:Cess is levied only for particular purpose and cannot be used for any other
purpose.

All the taxes collected by the government usually go into the Consolidated Fund of India
(CFI)which can be spent on any legitimate activity. But the collections from a cess are required
to be kept outside of the CFI to be spent only on the specific purpose for which it was levied

8.7. Arrange the following in decreasing order of their contribution to the Gross Tax Revenue.(Replace this as
GST has been introduced and they may not ask this question)

1. Corporation Tax

2. Income Tax

3. Custom Duty

a) 1-2-3

b) 2-3-1

c) 2-1-3

d) 3-1-2

Answer: A

Explanation: Corporation tax- 32%

Income tax- 22%

Custom Duty- 14%

Union Excise duty- 16%


Service Tax- 15%

Other taxes- 1%

9.8. Consider the following regarding Proportional Taxation:

1.It is a tax in which the rate of taxation is fixed which does not change as the taxable amount
increases or decreases.

2. It makes consumer spending more sensitive to fluctuations in GDP.

Which of the above statements is/are right?

a) 1 only

b) 2 only

c) Both 1 and 2

d) None of the above’

Answer: A

I don’t understand this explanation. Pl change it

Explanation:A proportional tax is a tax in which the rate of taxation is fixed which does not Formatted: Highlight
change as the taxable amount increases or decreases. This type of tax increases with increase
in income. While a regressive tax is a tax imposed in such a manner that the tax rate decreases
as the amount subject to taxation increases.

The proportional income tax acts as an automatic stabiliser - a shock absorber because it
makes disposable income, and thus consumer spending, less sensitive to fluctuations in
GDP.When GDP rises, disposable income also rises but by less than the rise in GDP because a
part of it is siphoned off as taxes. This helps limit the upward fluctuation in consumption
spending. During a recession when GDP falls, disposable income falls less sharply, and
consumption does not drop as much as it otherwise would have fallen had the tax liability been
fixed. This reduces the fall in aggregate demand and stabilises the economy.

10.9. Consider the following regarding Commodities transaction tax:

1. It is levied on transactions done on the domestic commodity derivatives exchanges except


agricultural commodities.

2. It is an indirect tax.

Which of the above statements is/are right?

a) 1 only

b) 2 only
c) Both 1 and 2

d) None of the above

Answer:A

Explanation:Commodities transaction tax (CTT) is a tax similar to Securities Transaction


Tax(STT), levied in India, on transactions done on the domestic commodity derivatives
exchanges except agricultural commodities.

It is considered as a direct tax.

11.10. With reference to the Micro, Small and Medium Enterprises in India, consider the following
statements:

1. Micro, Small & Medium Enterprises Development Act, 2006 (MSMED Act) provides for
classification of enterprises based on their investment size and the nature of the activity
undertaken by that enterprise
2. Government has reserved certain items to be manufactured exclusively by MSME sector.
Which of the statements given above is/are correct?

a) 1 only
b) 2 only
c) Both 1 and 2
d) Neither 1 nor 2
Answer: A

Explanation: Micro, Small & Medium Enterprises Development Act, 2006 (MSMED Act)
provides for classification of enterprises based on their investment size and the nature of the
activity undertaken by that enterprise. As per MSMED Act, enterprises are classified into two
categories - manufacturing enterprises and service enterprises

There is no item reserved for exclusive manufacture in the micro small and medium scale sector
unlike such a provision that existed during during pre-liberalisation phase. In 2015 Government
has de-reserved the remaining 20 items from the erstwhile list of items reserved for exclusive
manufacture in MSMEs.

12.11. For which of the following industries is Industrial Licensing compulsory?

1. Distillation and brewing of alcoholic drinks.

2. Railway transport

3. Drugs and Pharmaceuticals

4. Tobacco
5. Electronic Aerospace and Defence equipment.

a) 1, 3 , 4 and 5

b) 1 and 4

c) 2, 4 and 5

d) 3, 4 and 4

Answer: A

Explanation: Railway Transport and Atomic Energy are reserved for Public Sector. Issue of
Licencing, therefore, does not arise. The list of items covered under compulsory licensing under
the Industries (Development & Regulation) Act, 1951 is reviewed on an ongoing basis. At
present, only five industries are under compulsory licensing mainly on account of environmental,
safety and strategic considerations.

They are:
1. Distillation and brewing of alcoholic drinks
2. Cigars and cigarettes of tobacco and manufactured tobacco substitutes.
3. Electronic Aerospace and defense equipment: all types.
4. Industrial explosives including detonating fuses, safety fuses, gun powder, nitrocellulose and
matches.
5. Specified Hazardous chemicals i.e. (i) Hydrocyanic acid and its derivatives, (ii) Phosgene and its
derivatives and (iii) Isocyanates &diisocyanates of hydrocarbon, not elsewhere specified(example
Methyl isocyanate)

13.12. Which of the following is/ are recent policy initiatives taken by the Government of India

to promote export of services?

1. Services Exports from India Scheme

2. Global Exhibitions on Services

3. Services Conclaves

a) 1 and 3 only

b) 2 only

c) 1 and 2 only

d) 1, 2 and 3

Answer: D
Explanation: The government has taken a number of policy initiatives to promote services
exports which include the Service Exports from India Scheme (SEIS) (part of new Foreign Trade
Policy of 2015) for increasing exports of notified services from India; organizing Global
Exhibitions on Services (GES) and services conclaves to augment exports of various services
and position India as a key player in world services trade; besides some initiatives in sectors like
tourism and shipping.

14.13. Consider the following regarding Hybrid Annuity Model :

1. As per the design, the government will contribute to 40% of the project cost in the first five years
through annual payments
2. The developer has the right to collect toll.
3. HAM gives enough liquidity to the developer and the financial risk is shared by the government.
Which of the statements given above is/are correct?

a) 1 only

b) 1 and 2 only

c) 2 and 3 only

d) 1 and 3

Answer:D

Explanation: In India, the new HAM is a mix of BOT Annuity and EPC models. (Statement 1 is
correct). As per the design, the government will contribute to 40% of the project cost in the first
five years through annual payments (annuity). The remaining payment will be made on the
basis of the assets created and the performance of the developer. Here, hybrid annuity means
the first 40% payment is made as fixed amount in five equal installments whereas the remaining
60% is paid as variable annuity amount after the completion of the project depending upon the
value of assets created. As the government pays only 40%, during the construction stage, the
developer should find money for the remaining amount.

Here, he has to raise the remaining 60% in the form of equity or loans. There is no toll right for
the developer. Under HAM, Revenue collection would be the responsibility of the National
Highways Authority of India (NHAI). Advantage of HAM is that it gives enough liquidity to the
developer and the financial risk is shared by the government. While the private partner
continues to bear the construction and maintenance risks as in the case of BOT (toll) model, he
is required only to partly bear the financing risk. Government's policy is that the HAM will be
used in stalled projects where other models are not applicable.

15.14. Which of the following becomes the objective of Department of Investment and public Asset
Management (DIPAM)?
1. Improve corporate governance.
2. Realize the productive potential of CPSEs through improved efficiency and profitability.
3. CPSEs’ wealth should rest in the hands of the people.
4. Raise resources for the Government
a) 1,2,3
b) 1,2,4
c) 2,3,4
d) All of the above

Answer: D

Explanation:http://dipam.gov.in/frequently-asked-questions

Why does the Government disinvest CPSEs ?

Answer: The objectives of disinvestment are:

1. Improve corporate governance.


2. Realize the productive potential of CPSEs through improved efficiency and profitability.
3. CPSEs’ wealth should rest in the hands of the people.
4. Raise resources for the Government.

16.15. Consider the following statements:

1. ‘FDI’ means investment by non-resident entity/person resident outside India in the


capital of an Indian company .
2. ‘Downstream investment’ means indirect foreign investment, by an eligible Indian entity, into
another Indian company/LLP, by way of subscription or acquisition.
Which of the above statements is/are right?
a) 1 only
b) 2 only
c) Both 1 and 2
d) None of the above
Answer: C
Explanation: ‘FDI’ means investment by non-resident entity/person resident outside
India in the capital of an Indian company under Schedule 1 of Foreign Exchange
Management (Transfer or Issue of Security by a Person Resident Outside India)
Regulations, 2000
‘Downstream investment’ means indirect foreign investment, by an eligible Indian entity, into
another Indian company/LLP, by way of subscription or acquisition.

http://www.makeinindia.com/faqs
17.16. The new civil aviation policy relaxed which of the following rules: Change the question
accordingly.
a) ViabilityGapFunding(VGF)toairlineoperators
b) Relaxing 5:25 rule
c) Allowing foreign airlines to own 100% in Indian Carriers - This is allowed now
d) None of the above
Answer: A
Explanation:

1. The regional connectivity scheme provides for Viability Gap Funding(VGF)to airline
operators
2. it is 5:20 scheme
3. does not change pattern of ownership 49% fdi

http://pib.nic.in/newsite/PrintRelease.aspx?relid=146238

18.17. Project Insight of Government of India is related to:


a) Using funds for renovating medical infrastructure
b) Addressing gaps in gender participation in the workforce
c) Matching direct and indirect tax data to detect black money
d) Finding a fool proof mechanism to double farmers income in India
Answer : C
Explanation: income tax department and L & T infotech using big data to catch black
money

19.18. Consider the following statements with reference to Coastal Economic Zones
1. Coastal Economic Zones are spatial economic regions comprising a group of coastal
districts or districts with a strong linkage to ports in that region to tap into synergies with
the planned industrial corridor projects.
2. Cabinet has approved setting up of 14 mega CEZs under the National Perspective
Plan of the SagarmalaProgramme.
3. India’s first mega coastal economic zone (CEZ) – the Jawaharlal Nehru Port – is
located in Gujarat.
Which of the following statements is/are correct :
a) 1 and 2 only
b) 2 and 3 only
c) 1, 2 and 3 only
d) 1 and 3 only
Answer: A

Explanation: India’s first mega coastal economic zone (CEZ) – the Jawaharlal Nehru
Port – is located in Maharashtra.

Coastal Economic Zones are spatial economic regions comprising a group of coastal
districts or districts with a strong linkage to ports in that region to tap into synergies with
the planned industrial corridor projects.

Cabinet has approved setting up of 14 mega CEZs under the National Perspective Plan
of the SagarmalaProgramme.

20.19. Which of the following are considered as major challenge to Indian economy:
I. The US Federal Reserve's , intention to increase policy rates
II. Uncertainty around commodity prices, especially that of crude oil
III. Signs of retreat from globalisation of goods, services and people
Choose the correct option
a) 1 and 2 only
b) 2 and 3 only
c) 1, 2 and 3 only
d) 1 and 3 only
Answer: C

CHALLENGES IN 2017-18
World economy faces considerable uncertainty, in the aftermath of major economic and
political developments during the last year
The US Federal Reserve's , intention to increase policy rates in 2017, ma lead to lower capital
inflows and higher outflows from the emergineconomies
Uncertainty around commodity prices, especially that of crude oil, has implications for the
fiscal situation of emerging economies
Signs of retreat from globalisation of goods, services and people, as pressures for
protectionism are building up.

21.20. Consider the following statements


1. In the context of the newly launched GST, profiteering means that traders are
increasing the prices of the commodities when the GST Council reduces the tax rates
on commodities and services.
2. National Anti-Profiteering Authority (NAA) is the institutional mechanism under the
GST law to check the unfair profit-making activities by the trading community.
3. NAA’s core function is to ensure that the benefits of the reduction in GST rates on
goods or services made by the GST Council is passed on to the ultimate consumers by
way of a reduction in prices by traders.
Which of the above statements is/are correct?
a) 1 and 2 only
b) 2 and 3 only
c) 1 and 3 only
d) 1, 2 and 3
Answer: b
Explanation: Traders won’t increase the prices but they won’t the reducing the prices
while tax rates are reduced, thereby making profits.

22.21. As per Micro, Small and Medium Enterprises Development Act, 2006, micro enterprises
are those with investments:
a) < 25 lakhs
b) 25 lakhs to 5 crores
c) 5 crores to 10 crores
d) >10 crores
Answer: a.

23.22. The term “Open Acreage Licensing Policy” refers to


a) Exploration policy of hydrocarbons
b) Exploration policy of gold
c) Exploration policy of deep oceans
d) Allocation of land to SEZs
Answer: A

Explanation: OALP gives an option to a company to select the exploration


blocks on its own, without waiting for the formal bid round from the
Government.
● Under OALP, a bidder intending to explore hydrocarbons may apply to the

Government seeking exploration of any new block which was not already
covered by exploration.
● The Government will examine the interest and if it is suitable for award, then

the govt will call for competitive bids after obtaining necessary environmental
and other clearances.
● OALP was introduced as part of the new fiscal regime in exploration

sector called Hydrocarbon Exploration and Licensing Policy


(HELP).
● So that, it will enable a faster survey and coverage of the available

geographical area which has potential for oil and gas discovery.
● Successful implementation of OALP requires building of National Data

Repository on geo-scientific data

24.23. Which of the following reasons necessitated the intervention of State in the industrial
development after independence?
1. The private sector lacked the huge capital required for setting up of heavy industries.
2. The private players had less incentive to invest in industrialisation due to the low
demand for industrial goods.
3. The need to maximise the profit necessitated State intervention.
Which of the following statements are correct:
a) 1 only
b) 1 and 2 only
c) 2 and 3 only
d) 1, 2 and 3
Answer : D

25.24. With reference to Infrastructure Investment Trusts (InvITs), consider the following
statements:
1. They enable investments into the infrastructure sector by pooling small sums of
money from multiple investors.
2. Investments through InvITs can only be made in PPP project.
3. These are regulated by SEBI.
Which of the statements given above is/are correct?
a) 1 and 3 only
b) 2 only
c) 2 and 3 only
d) 1, 2 and 3
Answer : A
Infrastructure Investment Trusts (InvITs) are mutual fund like institutions that enable
investments into the infrastructure sector by pooling small sums of money from
multitude of individual investors for directly investing in infrastructure so as to return a
portion of the income (after deducting expenditures) to unit holders of InvITs, who
pooled in the money.

InvITs can invest in any infrastructure projects, either directly or through a special
purpose vehicle (SPV). In case of Public Private Partnership (PPP) projects, such
investments can only be through SPV.

InvITs are regulated by the securities market regulator in India- Securities and
Exchange Board of India

26.25. The components of India’s Foreign Exchange reserves includes


1) Foreign currency asset
2) Gold
3) Special drawing rights
4) Reserve tranche position

a) 1,3 and 4
b) 1,2 and 3
c) 1 and 3 only
d) All of the above
Ans : d
Solution :The components of India’s Foreign Exchange Reserves include Foreign
currency assets (FCAs), Special Drawing Rights (SDRs), Gold and RBI’s Reserve
position with International Monetary Fund (IMF).
FCAs forms major part of the overall reserves.

27.26. In the context of balance of payment , Which of the following items will not come under
current account
a) trade balance
b) imports
c) invisibles
d) short term debt
Ans : d
Solution : Current accounts consists of imports, exports, trade balance and invisibles (
Services, transfer and income ). Short term debt will come under capital account in
balance of payment.

28.27. There has been a debate over India’s optimum level of the forex reserves for
long time.
In this aspect , which of the following is adopted by RBI for exchange rate risk
1) Fixed exchange rate
2) Fixed foreign exchange reserves
3) Intervention in the forex market to reduce volatility.
Choose the correct option:
a) 1 and 2 only
b) 2 and 3 only
c) 3 only
d) 1, 2 and 3
Ans : c
Solution :There has been a debate over India’s optimum level of the forex reserves. The
RBI is aware of the downside risks to the exchange rate,as is reflected by its action of
buying the US dollar. Officially, the RBItargets neithera particular exchange rate nor
foreign exchange reserves, andmaintains such interventions by it to just reduce
volatilityin the forex market.

29.28. In order to curtail volatile nature of rupee against the dollar, RBI purchases dollars from
the market , it leads to
1) infusion of rupee into the system
2) increases inflationary effect on the economy
3) absorps rupee from the system
4) decreases inflationary effect on the economy
Choose the correct option :
a) 1 and 4 only
b) 2 and 3 only
c) 1 and 2 only
d) 3 and 4 only
Ans : c
Solution : Forex reserves act as insurance when the rupee tends to be volatile against
the dollar . When RBI purchases dollars from the market, it leads to infusion of rupee
into the system which leaves inflationary effect on the economy.

30.29. Consider the following statements regarding external debt :


1) The maturity pattern of India’s external debt indicated dominance of short term
borrowing.
2) Cross country comparison of external debt indicates that India continues to be among
the less vulnerable countries .
Which of the following statements given above is/are correct?
a) 1 only
b) 2only
c) Both 1 and 2
d) Neither 1 nor 2
Ans : b
Solution : The maturity pattern of India’s external debt indicated dominance of long-
term borrowing – long term debt accounted for 83.2 per cent ( rest 16.8 per cent being
short-term i.e., of upto one year maturity period.
Cross- country comparison of external debt indicates that India continues to be among
the less vulnerable countries and its key indicators compare well with other indebted
countries of the developing world.

Day 2 :
31.30. Which of the following define the “ floating exchange rate system “
a) it is an exchange rate system of a particular currency fixed by the IMF keeping the
currency in front of a basket of world currencies.
b) it is an exchange rate system , in which a domestic currency is left free to float
against a number of foreign currencies in its foreign exchange market.
c) it is an exchange rate system in which government of the economy attempts to affect
the exchange rate directly by buying or selling foreign currencies or indirectly through
monetary policy.
d) None of the above
Ans : b
Solution : Fixed exchange rate system - it is an exchange rate system of a particular
currency fixed by the IMF keeping the currency in front of a basket of world currencies.
Floating exchange rate system - it is an exchange rate system , in which a domestic
currency is left free to float against a number of foreign currencies in its foreign
exchange market and determine its own value.
Managed exchange rate system - it is an exchange rate system in which government of
the economy attempts to affect the exchange rate directly by buying or selling foreign
currencies or indirectly through monetary policy.

32.31. Consider the following statements regarding exchange rate in India


1) India moved to the floating currency regime with its own method which is known as
the ‘dual exchange rate’
2) In this dual exchange rate system, RBI is not allowed to intervene in the forex market.
Which of the following statements given above is/are correct?
a) 1 only
b) 2 ony
c) Both 1 and 2
d) Neither 1 nor 2
Ans : a
Solution : In 1992–93 financial year, India moved to the floating currency regime
with its own method which is known as the ‘dual exchange rate’. There are
two exchange rates for rupee, one is the ‘official rate’ and the other is the
‘market rate’. Here the point should be noted that it is the everyday’s
changing market-based exchange rate of rupee which affects the official
exchange rate and not the other way round. But the RBI may intervene in the
forex market via the demand and supply of rupee or the foreign currencies.
Another point which should be kept in mind is that none of the economies
have till date followed an ideal free-floating exchange rate. They require
some mechanism to intervene in the foreign exchange market because this is
a highly speculative market.

33.32. Which of the following statements define the term “revaluation” with respect to
foreign exchange market ?
a) when domestic currency loses its value in front of a foreign currency and it is market
driven
b) when domestic currency is cut down by its government against any foreign currency.
c) when a government increasing the exchange rate of its domestic currency against
any foreign currency.
d) when a free floating domestic currency increases its value against the value of a
foreign currency.
Ans : c
Solution : Depreciation - when domestic currency loses its value in front of a foreign
currency and it is market driven
Devaluation - when exchange rate of a domestic currency is cut down by its
government against any foreign currency. It is also called as official depreciation.
Revaluation - when a government increasing the exchange rate of its domestic
currency against any foreign currency. It is also called as official appreciation.
Appreciation - when a free floating domestic currency increases its value against the
value of a foreign currency.

34.33. Which of the following transaction in foreign currency is considered as capital in


capital account ?
1) external borrowings and lending
2) foreign currency deposits of banks
3) Security market investment of Qualified Foreign Investors QFIs
4) Private remittances and transfers
5) FDI
Choose the correct option :
a) 1, 2 and 5 only
b) 1, 2, 3 and 5 only
c) 1, 2 and 3 only
d) 2, 3 and 4 only
Ans : b
Solution :Every government of the world maintains a capital account, which shows
thecapital kind of transactions of the economy with outside economies. Every
transaction in foreign currency (inflow or outflow) considered as capital is
shown in this account—external lending and borrowing, foreign currency
deposits of banks, external bonds issued by the Government of India, FDI,PIS and
security market investment of the QFIs (Rupee is fully convertible in
this case ) .

There is no deficit or surplus in this account like the current account .


Private remittances and transfers come under current account.

35.34. Consider the following statements regarding BoP crisis


1) A negative BoP is unfavourable for an economy if only the economy lacks the means
to fill the gap of negativity.
2) If the forex reserves are capable of fulfilling the negativity created by the BoP, it is
known as a BoP crisis.
Which of the following statements given above is/are correct?
a) 1 only
b) 2 ony
c) Both 1 and 2
d) Neither 1 nor 2
Ans : a
Solution : The outcome of the total transactions of an economy with the outside world in
one year is known as the balance of payment (BoP) of the economy.
Basically, it is the net outcome of the current and capital accounts of an
economy. It might be favourable or unfavourable for the economy. However, negativity
of the BoP does not mean it is unfavourable. A negative BoP is
unfavourable for an economy if only the economy lacks the means to fill the
gap of negativity.

Ifthere is a positive outcome at the end of the year, the money is automatically
transferred to the foreign exchange reserves of the economy. And if there is
any negative outcome, the same foreign exchange is drawn from the
country’s forex reserves. If the forex reserves are not capable of fulfilling the
negativity created by the BoP, it is known as a BoP crisis and the economy
tries different means to solve the crisis in which going for forex help from the
IMF is the last resort.

Day 3 :

36.35. Which of the following is the correct sequence in increasing degree of


economic integration between member countries?
a) preferential trade agreement – free trade agreement – economic union – custom
union – common market
b) free trade agreement – preferential trade agreement – economic union – common
market – custom union
c) preferential trade agreement – free trade agreement – cutom union – economic union
– common market
d) preferential trade agreement – free trade agreement – custom union – common
market – economic union.

Ans : d

Solution : The correct sequence of increasing degree of economic integration between


member countries are preferential trade agreement – free trade agreement – custom
union – common market – economic union.
37.36. Which of the following items given below come under tariff barrierare related to
“barriers to trade”?
1) Quota
2) production subsidies
3) export subsidies
4) health , sanitary and safety regulations.
Choose the correct option:
a) 1, 2 and 3 only
b) 1, 2 and 4 only
c) 2 and 4 only
d) None of the above
Ans : d
Solution : All the four items ( i.e., Quota, production subsidies, export subsidies, health ,
sanitary and safety regulations) will come under non tariff barriers.
Tariff barriers are barriers by imposing duty on import and export of goods.The purpose
for tariff on imports and exports is to generate resource to government and to obstruct
free flow of trade.
The instruments and executive operations that obstruct free flow of trade other than
tariff is called non tariff barriers.

38.37. Consider the following statements :


1) SDR is a reserve created by International Monetary Fund (IMF) to help countries that
have BoP crisis.
2) In India , Special Drawing Rights holding is held with the Finance Ministry.
Which of the following statements given above is/are correct?
a) 1 only
b) 2 ony
c) Both 1 and 2
d) Neither 1 nor 2
Ans : a
Solution :SDR is a reserve created by International Monetary Fund (IMF) to help
countries that have BoP crisis. The member countries have to contribute to this account.
The contribution is in proportion of their IMF quota( membership fee) . It is held with the
government or the Central bank of the member countries. In India, it is with RBI’s
exchange reserve.

39.38. Which of the following given below is/ are considered as Foreign Portfolio
Investments ?
1) Foreign Institution Investment
2) Depository receipts
3) Offshore funds
4) shares acquired by way of IPO
Choose the correct option:
a) 1 only
b) 1 and 2 only
c) 1, 2 and 3 only
d) 1, 2, 3 and 4
Ans : c
Solution : Investment through stock exchange that is through secondary market is called
portfolio investments. Portfolio investment refers to investment in various financial
instruments like shares, debentures of a company through secondary market. There are
three major types of portfolio investment. They are
1) Foreign Institution Investment
2) Depository receipts
3) Offshore funds
Shares acquired by way of IPO will come under Direct Investment.

40.39. With respect to NRI deposits , deposits are held in which of the following foreign
currencies ?
1) US Dollar
2) Pound Sterling
3) Japanese Yen
4) Chinese Renminbi
5) Canadian Dollar
Choose the correct option:
a) 1, 2, 3 and 5 only
b) 1, 2, and 3 only
c) 1, 2 and 5 only
d) 1 and 2 only
Ans : a
Solution : NRI deposits are held in the following foreign currencies. They are
1) US Dollar
2) Pound Sterling
3) Japanese Yen
4) Australian Dollar
5) Canadian Dollar
Only term deposits of one to three years maturity is allowed.

Day 4 :

41.40. Which are the sectors where foreign investment is prohibited?


1) Gambling and betting including casinos
2) Chit funds
3) Nidhi companies
4) Real estate business or construction of farm houses
5) Manufacturing of cigars or of tobacco substitutes.
Choose the correct option :
a) 1, 4 and 5 only
b) 1 and 5 only
c) 1, 2 and 5 only
d) All the above
Ans : d

Solution : Foreign investment is prohibited in the following sectors:

i. Lottery Business including Government / private lottery, online lotteries, etc.


ii. Gambling and Betting including casinos etc.
iii. Chit funds
iv. Nidhi company
v. Trading in Transferable Development Rights (TDRs)
vi. Real Estate Business or Construction of Farm Houses
vii. Manufacturing of Cigars, cheroots, cigarillos and cigarettes, of tobacco or of tobacco
substitutes
viii. Activities / sectors not open to private sector investment e.g. (I) Atomic energy and (II)
Railway operations (other than permitted activities mentioned in entry 18 of Annex B).

Note: Foreign technology collaboration in any form including licensing for franchise,
trademark, brand name, management contract is also prohibited for Lottery Business
and Gambling and Betting activities.

42.41. Which of the following accounts that can be opened in India in a foreign
currency by a non-resident?

1) NRE-Non-Resident (External) Rupee Account

2) FCNR(B) Foreign Currency (Non-Resident) Account (Banks) Scheme

3) NRONon-Resident Ordinary Rupee Account

Choose the correct option :

a) 2 only

b) 3 only

c) 1 only

d) 1 and 3 only
Ans : a
Solution :Foreign Currency (Non-Resident) Account (Banks) Scheme [FCNR (B)
Account] is a non-resident account that can be opened in India in foreign currency
which is freely convertible. Non-Resident (External) Rupee Account Scheme
[NRE Account] and Non-Resident Ordinary Rupee Account Scheme [NRO Account] are
also non-resident account that can be opened in India in rupee.

43.42. Which of the following organisations is/ are eligible for issuance of Rupee
denominated bonds overseas?
1) Corporate registered under companies act
2) Corporate created out of specific act of the Parliament
3) Indian banks
4) Real estate Investment Trust
Choose the correct option:
a) 1 and 2 only
b) 1, 2 and 4 only
c) 1, 2, 3 and 4
d) 1 only
Ans : c
Solution : Any corporate (entity registered as a company under the Companies Act,
1956/ 2013) or body corporate (entity specially created out of a specific act of the
Parliament) and Indian banks are eligible to issue Rupee denominated bonds overseas.
Real Estate Investment Trusts (REITs) and Infrastructure Investment Trusts (InvITs)
coming under the regulatory jurisdiction of the Securities and Exchange Board of India
(SEBI) are also eligible. Other resident entities like Limited Liability Partnerships and
Partnership firms, etc. are also not eligible to issue these bonds.

44.43. Which one of the following is the nature of convertibility of the Indian economy?
a) Convertibility at capital account
b) Convertibility at current account
c) Freely convertible
d) Convertibility at trade account
Ans : b

Solution : When Partial convertibility of Rupee on current account was introduced 1992,
government had announced its intention to introduce the full convertibility on the current
account in 3 to 5 years. The full convertibility means no RBI dictated rates and there is a
unified market determined exchange rate regime. Encouraged with the success of the
LERMS, the government introduced the full convertibility of Rupee in Trade
account(means only merchandise trade no service trade)from March 1993 onwards.
With this the dual exchange rate system got automatically abolished and LERMS was
now based upon the open market exchange. The full convertibility of Rupee was
followed by stability in the Rupee Rate in the next many months coming up.

The above full convertibility was introduced on Trade account. The Government wanted
to introduce the Full convertibility of Rupee on Current account (means invisible also
included). In August 1994, the Government of India declared full convertibility of Rupee
on Current account with announcing some relaxations as per requirements of the Article
VIII of the IMF.

45.44. Which one of the following is/are non-debt flow of capital between different
countries?

1) Foreign Direct Investment

2) Foreign portfolio Investment

3) Commercial borrowings

4) Short term borrowing from IMF

Choose the correct option:

a) 1 and 2 only

b) 1, 2 and 3 only

c) All the above

d) None of the above

Ans : a

Solution : Non-debt flow of capital between different countries are

1) Foreign Direct Investment

2) Foreign portfolio Investment

Commercial borrowings and Short term borrowing from IMF are considered as debt flow
of capital between different countries.

Day 5 :
46.45. Which of the following countries are part of a Regional Comprehensive Economic
Partnership Agreement (RCEP) along with ASEAN countries

1) Australia

2) China

3) India

4) Japan

5) Sri Lanka

Choose the correct option:

a) 1, 2, 3 and 4 only

b) 1, 2, 3 and 5 only

c) 2, 3, 4 and 5 only

d) All the above mentioned countries

Ans : a

Solution :RCEP Agreement among ASEAN + 6 FTA Partners (Australia, China, India,
Japan, South Korea and New Zealand). Based on the Declaration of the Leaders during
the ASEAN Summit in November, 2012, negotiations for a Regional Comprehensive
Economic Partnership Agreement (RCEP) between the 10 ASEAN member states and
its 6 FTA partners commenced in May, 2013. The 3rd Intersessional RCEP Ministerial
was recently concluded in Hanoi from 21-22 May, 2017. India will be hosting the 19th Formatted: Highlight

RCEP Round from 18-28 July, 2017 in Hyderabad. (This period is already over). Is it
correct? The negotiations cover a number of areas like trade in goods, services,
investment, intellectual property, economic & technical cooperation, competition and
legal & institutional issues.

47.46. Consider the following statements:


1) The new Foreign Trade Policy (2015-20) launched on 1st April, 2015 links rules,
procedures and incentives for exports and imports with other initiatives
2) A new scheme Trade Infrastructure for Export Scheme (TIES) has been approved to
be implemented from 2017-18 for 3 years.
Which of the following statements given above is/are correct?
e) a) 1 only Formatted: No bullets or numbering
f) b) 2 only
g) c) Both 1 and 2
h) d) Neither 1 nor 2
Ans : c
Solution : The new Foreign Trade Policy (2015-20) launched on 1st April, 2015
links rules, procedures and incentives for exports and imports with other
initiatives such as “Make in India”, “Digital India” and “Skills India”. It consolidates
5 different incentive schemes under the earlier policy for rewarding merchandise
exports into a single scheme, namely the Merchandise Exports from India Scheme
(MEIS). For the services sector, the Services Exports from India Scheme (SEIS) has
been introduced replacing the Served from India Scheme. The Interest Equalisation
scheme on pre and post shipment rupee export credit was also approved by the
Cabinet Committee on Economic Affairs (CCEA) on 18th November 2015 w.e.f. 1st
April 2015 for 5 years and will be evaluated after three years. A new scheme Trade
Infrastructure for Export Scheme (TIES) has been approved to be implemented
from 2017-18 for 3 years. Besides many trade facilitation measures have also been
taken like reducing the number of documents, introducing simplified IEC (Importer
Exporter Code) from 1st April 2016, doing away with the issuance of physical copy of
IEC, sharing export realization data with states and encouraging states to prepare their
export strategies resulting in 17 states preparing their export strategies, simplifying
AayatNiryatforms, etc.

48.47. The largest portion of India’s External debt is contributed from which of the
following component?
a) commercial borrowing
b) NRI deposits
c) Multilateral borrowing
d) Bilateral borrowing
Ans : a
Solution : The largest portion of India’s External debt is contributed from commercial
borrowing followed by NRI deposits, Multilateral borrowings and bilateral borrowing.
All above mentioned components will fall under Long term debt.

49.48. International Debt Statistics is published by which one of the following


organizations?
a) International Monetary Fund
b) World Bank
c) World Economic Forum
d) World Trade Organization
Ans : b
Solution : International Debt Statistics is published by World Bank. International Debt
Statistics (IDS) (formerly Global Development Finance) provides statistical tables
showing the external debt of 125 developing countries that report public and publicly
guaranteed external debt to the World Bank's Debtor Reporting System (DRS).

50.49. Which of the following statements define the term “heated currency”?
a) currency which is under pressure of depreciationdue to a hard currency exit
b) currency that is easily available in any economy in its forex market.
c) currency is exiting any economy at a fast pacefor the time
d) currency of the world, which has thehighest level of liquidity.
Ans : a
Solution :

HEATED CURRENCY

This term is used to denote the domestic currency which is under pressure (heat) of
depreciation due to a hard currency’s high tendency of exiting the economy. Also
known as currency under heat or under hammering.

HOT CURRENCY

It is the term for the Forex market and is the temporary name for any Hard currency. If
any Hard currency is exiting any economy at a fast pace for the time, the Hard
currency is said to be Hot currency.

SOFT CURRENCY

Soft currency you can say is just the opposite of Hard currency. It is the currency that
is easily available in any economy in its forex market.

For example, Indian Rupee is the Soft currency in the Indian Forex market.

HARD CURRENCY

Hard Currency is the international currency in which the highest faith is shown and is
needed by every economy.You can say it is basically the strongest currency of
the world, which has the highest level of liquidity.
51.50. Which of the following are true ?

i)The term Infrastructure is seen in the purview of economy only.


ii)Improvements inhealth infrastructurehave a large scale impact in reducing morbidity.
Choose the appropriate code
a)i only
b)ii only
c)Bothi& ii
d)Neither I nor ii

Solution : B
Infrastructure not only includes economic infrastructure like road,rail, airport. Social
Infrastructure like health care, sanitation, water supply are also part of Infrastructure.
2nd statement is the interlinking statement between health infrastructure and health indicator.

52.51. Which of the following are correct regarding rural infrastructure in India.

i)The census 2001shows that in ruralIndia only 56 percent householdshave an


electricityconnection and 43 per cent still use kerosene.
ii)Pradhan Mantri Gram SadakYojana aims to Provide uninterrupted electricity supply to rural areas.
Choose the appropriate code
a)i only
b)ii only
c)Bothi& ii
d)Neither I nor ii

Solution :A
The census 2001shows that in ruralIndia only 56 percent householdshave an
electricityconnection and 43per cent still usekerosene. About90 per cent of therural
householdsuse bio-fuels forcooking. Tap wateravailability islimited to only 24per cent
ruralhouseholds. About76 per cent of thepopulation drinkswater from opensources such as
wells, tanks, ponds,lakes, rivers, canals, etc. Access toimproved sanitation in rural areas
wasonly 20 per cent.
PM gram SadakYojana is to provide road connectivity to rural areas implemented by
Rural Development Ministry.

53.52. Which of the following are true regarding the following PPP model

i)BOT (Build Operate and Transfer) : Private partner is responsible to design, build, operate
(during the contracted period) and transfer back the facility to the public sector.
ii)Build Own Operate : Ownership of the newly built facility will rest with the Government.
Choose the appropriate code
a)i only
b)ii only
c)Bothi& ii
d)Neither I nor ii
Solution a
Build Operate and Transfer (BOT): This is the simple and conventional PPP model where the
private partner is responsible to design, build, operate (during the contracted period) and
transfer back the facility to the public sector. Role of the private sector partner is to bring the
finance for the project and take the responsibility to construct and maintain it. In return, the
public sector will allow it to collect revenue from the users. The national highway projects
contracted out by NHAI under PPP mode is a major example for the BOT model.
Build-Own-Operate (BOO): This is a variant of the BOT and the difference is that the ownership
of the newly built facility will rest with the private party here. The public sector partner agrees to
‘purchase’ the goods and services produced by the project on mutually agreed terms and
conditions.

54.53. Which of the following are incorrect regarding the following PPP model

i) Lease-Develop-Operate (LDO): Private sector entity retains ownership of the newly created
infrastructure facilityand receives payments in terms of a lease agreement with the private
sector.
ii) DBFO (Design, Build, Finance and Operate : Private party assumes the entire responsibility
for the design, construction, finance, and operate the project for the period of concession.
Choose the appropriate code
a)i only
b)ii only
c)Bothi& ii
d)Neither I nor ii

Solution a
Lease-Develop-Operate (LDO): Here, the government or the public sector entity retains
ownership of the newly created infrastructure facility and receives payments in terms of a lease
agreement with the private promoter. This approach is mostly followed in the development of
airport facilities.
DBFO (Design, Build, Finance and Operate): In this model, the private party assumes the entire
responsibility for the design, construction, finance, and operate the project for the period of
concession.

55.54. Which of the following are components of Infrastructure development project SAGARMALA.

i)Port Modernization & New Port Development


ii) Port Connectivity Enhancement
iii) Port-linked Industrialization
iv) Coastal Community Development
Choose the appropriate code
a)I,ii,iii only
b)I,ii only
c)I,iii only
d)I,ii,iii,iv only
Solution d
Components of SagarmalaProgramme are:

 Port Modernization & New Port Development: De-bottlenecking and capacity expansion of existing ports and
development of new greenfield ports
 Port Connectivity Enhancement: Enhancing the connectivity of the ports to the hinterland, optimizing cost and time
of cargo movement through multi-modal logistics solutions including domestic waterways (inland water transport and
coastal shipping)
 Port-linked Industrialization: Developing port-proximate industrial clusters and Coastal Economic Zones to reduce
logistics cost and time of EXIM and domestic cargo
 Coastal Community Development: Promoting sustainable development of coastal communities through skill
development & livelihood generation activities, fisheries development, coastal tourism etc.

56.55. Which of the following is the objective of Regional Connectivity scheme.

a) To connect underserved airports to key airports through flights.


b) To Provide Internet Connectivity to Rural areas.
c) To provide seamless uninterrupted Power supply of every village in India.
d) Scheme aims to connect the various educational institutes from different regions of India.

Solution a
Airports Authority of India (AAI) under Regional Connectivity Scheme will connect 22 airports
under regional connectivity scheme in the first phase.
22 airports include one is in Andaman Nicobar, three in Assam, two each in Gujarat, Uttar
Pradesh, Punjab and Rajasthan.
Under RCS plans are to connect these underserved airports to key airports through flights that
will cost Rs 2,500 for per hour flight. RCS envisages to provide subsidy to airlines to offer these
fares. AAI would invest Rs 17,500 crore in upgrading airport infrastructure over a period till
2019-20.
57.56. Which of the following are true regarding Dedicated Freight corridor.

i) Dedicated Freight Corridor (DFC) ply’s in in two longroad routes namely, the Eastern and
Western freight corridors.
ii) Eastern Dedicated Freight Corridor stretching from Ludhiana to Dankuni in West Bengal and
the Western Dedicated Freight Corridor from Jawaharlal Nehru Port in Mumbai (Maharashtra) to
Dadri.
Choose the appropriate code
a)i only
b)ii only
c)Bothi& ii
d)Neither I nor ii

Solution:C
Under the Eleventh Five Year Plan of India (2007–12), Ministry of Railways is constructing a
new Dedicated Freight Corridor (DFC) in two long routes namely, the Eastern and Western
freight corridors. The two routes covers a total length of 3,360 kilometres (2,090 mi), with the
Eastern Dedicated Freight Corridor stretching from Ludhiana in Punjab to Dankuni in West
Bengal and the Western Dedicated Freight Corridor from Jawaharlal Nehru Port in Mumbai
(Maharashtra) to Dadri in Uttar Pradesh. Upgrading of transportation technology, increase in
productivity and reduction in unit transportation cost are the focus areas for the project. Thus
statement 1 is wrong because it is not a roadway, it’s a Railway corridor.

DFCCIL has been designated by Government of India as a `special purpose vehicle`, and has
been created to undertake planning & development, mobilization of financial resources and
construction, maintenance and operation of the Dedicated Freight Corridors.

58.57. Consider the following statements are true.

i) Asia's first EPZ (Export Processing zone) was set up in India in 1965.
ii) The Nodal agency for Special Economic zones in India is Ministry of Finance.
Choose the appropriate code
a)i only
b)ii only
c)Bothi& ii
d)Neither I nor ii
Solution a
Statement 1 is right . Asia’s first EPZ (Export Processing zone) was set up in kandla (Gujarat-
India) in 1965.
Special Economic zones are under the ageis of Ministry of Commerce and Industry.

59.58. Which of the following are correct regarding the terms below.

i) A JV (Joint Venture) is created as a separate corporate entity to implement a particular


project.
ii)An SPV (Special Purpose Vehicle is an entity created through equity participation of multiple
firms to do business in a particular area.
Choose the appropriate code
a)i only
b)ii only
c)Bothi& ii
d)Neither I nor ii
Solution d
An SPV is created as a separate corporate entity to implement a particular project. A JV is an
entity created through equity participation of multiple firms to do business in a particular area.

SPVs/SPEs may be formed through limited partnerships, trusts, corporations, limited liability corporations
or other entities. An SPV/SPE may be designed for independent ownership, management and funding of
a company; as protection of a project from operational or insolvency issues; or for creating a synthetic
lease that is expensed on the company’s income statement rather than recorded as a liability on the
balance sheet. They help companies securitize assets, create joint ventures, isolate corporate assets or
perform other financial transactions.

A joint venture (JV) is a business arrangement in which two or more parties agree to pool their resources
for the purpose of accomplishing a specific task. This task can be a new project or any other business
activity. In a joint venture (JV), each of the participants is responsible for profits, losses and costs
associated with it. However, the venture is its own entity, separate and apart from the partic ipants' other
business interests.

60.59. Which of the following are incorrect regarding Health Infrastructure.

i)In recent times, private sector has been playing a dominant role in medical education and
training, medical technology and diagnostics, manufacture and sale of pharmaceuticals.
ii)National Health Policy 2017,aims to raise public healthcare expenditure to 2.5% of GDP.
Choose the appropriate code
a)i only
b)ii only
c)Bothi& ii
d)Neither I nor ii
Solution d
Both the statements are right. Incorrect statement is asked.
Highlights of National Health Policy, 2017
It aims to raise public healthcare expenditure to 2.5% of GDP from current 1.4%, with more than
two-thirds of those resources going towards primary healthcare. It envisages providing a larger
package of assured comprehensive primary healthcare through the ‘Health and Wellness
Centers’.
It is a comprehensive package that will include care for major non-communicable diseases
(NCDs), geriatric healthcare, mental health, palliative care and rehabilitative care services.
It proposes free diagnostics, free drugs and free emergency and essential healthcare services
in all public hospitals in order to provide healthcare services in all public hospitals in order to
provide healthcare access and financial protection....
Poverty, Unemployment

61.60. Consider the following statements about Socio Economic Caste Census
2011.
1)Ministry of Rural Development was the overall coordinator of the
exercise.
2) This is the first time such a comprehensive exercise has been carried
out for both rural and urban India.
Choose the correct statements.
a. 1 only
b. 2 only
c. Both 1 and 2
d. None of the above
Answer: c
Explanation:
The Ministry of Rural Development Government of India, commenced the
Socio Economic and Caste Census (SECC) 2011, in June 2011 through a
comprehensive door to door enumeration across the country. This is the
first time such a comprehensive exercise has been carried out for both rural
and urban India. It is also expected to generate information on a large
number of social and economic indicators relating to households across the
country.
The SECC, 2011 has the following three objectives:
 To enable households to be ranked based on their Socio- Economic status.
State Governments can then prepare a list of families living below the
poverty line

 To make available authentic information that will enable caste-wise


population enumeration of the country
 To make available authentic information regarding the socio economic
condition, and education status of various castes and sections of the
population

SECC 2011 has three census components which were conducted by three
separate authorities, but under the overall coordination of Department of
Rural Development in the Government of India:
Census in Rural Area has been conducted by the Department of Rural
Development.
Census in Urban areas is under the administrative jurisdiction of the
Ministry of Housing and Urban Poverty Alleviation.
Caste Census is under the administrative control of Ministry of Home
Affairs: Registrar General and Census Commissioner of India.
SECC data 2011 has been referred for analysis by an expert group,
headed by NITI Aayog Vice-Chairman Arvind Panagariya.[45] This expert
group is set up by the ministries of social justice and tribal development,
before making public.

62.61. India belongs to which of the following four human development categories
as per the Human Development Report 2016.
a)Very High Human Development
b) High Human Development
c) Medium Human Development
d) Low Human Development.
Answer: - c
Explanation:-
India was ranked 131 in the 2016 Human Development Index (HDI) among
the 188 countries. India scored 0.624 and was placed in medium human
development.
The index was unveiled recently as part of the Human Development Report
(HDR) 2016 titled Human Development for Everyone published by the
United Nations Development Program.
The Human Development Index (HDI) is a composite statistic of life
expectancy, education, and income per capita indicators. A country scores
higher HDI when the life expectancy at birth is longer, the education period
is longer, and the income per capita is higher. It is used to distinguish
whether the country is a developed, a developing or an underdeveloped
country. The index was developed in 1990 by Pakistani economist
MahbubulHaqand Indian economist Amartya Sen.

The UN report covers 185 member states of the United Nations (out of
193), along with Hong Kong and Palestine; 8 UN member states are not
included because of lack of data. The average HDI of regions of the World
and groups of countries are also included for comparison.
Countries fall into four broad human development categories: Very High
Human Development, High Human Development, Medium Human
Development and Low Human Development.
The national average HDI for India in 2008 was 0.467. By 2010, its average
HDI had risen to 0.519. UNDP, the sponsor of Human Development Index
methodology since 1990, reported India's HDI to be 0.554 for 2012, an
18% increase over its 2008 HDI. United Nations Declared India's HDI is
0.586 in 2014, an 5.77% increase over 2012. As for the year 2016, HDI for
India stood at 0.624.

63.62. Recently the Central Government has accepted recommendations to use


one of the following as the main instrument for identification of beneficiaries and
transferring of funds for social schemes in rural areas. Choose the correct option.
a) Socio Economic Caste Census 2011
b) Poverty Line
c) Multidimensional poverty index
d) None of the above

Answer:- a
Explanation:-
Government will adopt the Socio-Economic and Caste Census (SECC)
instead of the poverty line-based method to identify recipients for its pro-
poor schemes. The SECC 2011 ranks households based on their socio-
economic status to enable state governments to prepare a list of families
living below the poverty line. It also makes available information regarding
the socio-economic condition and education status of various castes and
sections of the population.
The Ministry of Rural Development will start using SECC 2011 data this
year for its National Social Assistance Programme to pay pension to rural
poor and National Rural Livelihood Mission. Both schemes use BPL data to
estimate the number of the poor. “BPL data tells us how many are poor and
SECC who are those poor… it is a more targeted and scientific approach in
ensuring the right person gets the benefit.
TheSumit Bose Committee formed to study the validity and efficiency of
the SECC 2011 data in identifying the poor recently submitted its report,
which is in favour of using the information for rural development schemes.
The committee also developed a formula based on deprivation parameters
to identify beneficiaries for specific schemes.
The greater the deprivation score, the higher will be the ranking of a
household for getting government assistance. The government is also
identifying workers under the national rural employment guarantee scheme
who have reported deprivation in the SECC study.
SECC data provides for automatic exclusion of beneficiaries on the basis of
14 parameters, automatic inclusion on five parameters and grading of
deprivation on the basis of seven criteria.

Where there is disparity in identifying beneficiaries using BPL certificates


and SECC data, the government is likely to correct the anomaly on the
basis of the census information.
SECC will have to prevail over poverty line in such a case as only one data
can be put to use and undeserving beneficiaries will have to be weeded
out

64.63. Consider the following statements regarding employment elasticity.


1) Employment elasticity is the percentage change in employment with 1%
change in economic growth.
2) A positive employment elasticity results in jobless growth.
Choose the correct statements.
a. 1 only
b. 2 only
c. Both 1 and 2
d. None of the above
Answer: a
Explanation:
Employment elasticity is a measure of the percentage change in
employment associated with a 1 percentage point change in economic
growth. The employment elasticity indicates the ability of an economy to
generate employment opportunities for its population as per cent of its
growth (development) process.
India has low or negative employment elasticity year-on-year basis and this
indicates that India’s growth is jobless growth. During 2004-14, India saw
some of the highest rates of gross domestic product (GDP) but it did not
translate into jobs. The number of workers did not only decrease in farm
sector but also manufacturing. The government explains this by quoting
that labour intensive technology has been replaced by capital intensive
technology and more and more labour force has been accommodated in
unorganized sectors or new jobs in the informal sector, with many others
went converted into successful small scale entrepreneurs. This is used to
explain that the highest employment elasticity has been shown by the
Construction and utilities sector (which includes energy, water and waste
management). These are the biggest job generators in our country.

65.64. On which of the following factors does sectoral employment elasticity


depend on?
a) Growth of a sector
b) labour intensiveness of that sector.
c) Both a and b
d) None of the above.
Answer:- c
It can be explained by differentiating between sectoral employment
elasticity and aggregate employment elasticity of the economy.
As per RBI report aggregate employment elasticity depends on three
factors: sectoral employment elasticity, relative growth of a sector and its
employment share.
So, even though manufacturing has the highest employment elasticity, its
overall impact has been muted because of slow increase in employment
share and low relative growth (ratio between growth of manufacturing and
overall economy). By the same logic, agriculture is having a large negative
impact on employment generation.
These facts underline the importance of programmes such as Make in India
to tackle India’s employment challenge.

66.65. Assertion: India is encouraging the apparel industry and MSME’s to tackle
the issue of jobless growth.
Reason: The above industries are Capital intensive industries.
a. Both assertion and reason are true, reason is correct explanation of
assertion.
b. Both assertion and reason are true but reason is not a correct explain
the assertion.
c. Assertion is true but reason is false.
d. Assertion is false but reason is true.

Answer:- c
Explanation:-
Within manufacturing, employment elasticity for the organised
manufacturing sector lies in the range of 0.4-0.5 for the 2000s according to
various estimates. Accordingly, apparel, furniture, leather products, motor
vehicles, rubber products and electrical equipment were identified as being
employment-intensive sectors during the 2000s. Therefore, the recent (post
8 November 2016) slowdown in the manufacturing, mining and construction
sectors has huge implications for employment generation in India
India's apparel sector has immense potential for employment generation. It
provides employment to over 45 million directly and 60 million people
indirectly according to Apparel Export Promotion Council estimates.
Alluding to the sector's importance for employment, a World Bank report
("Stitches to Riches") points out that a 10% rise in apparel prices in China
could help India create at least 1.2 million jobs.

Increasing wage costs in China and the country's move to higher value-
added exports are therefore expected to provide opportunities for labour-
intensive sectors (like apparel) in countries like India.
Migration of labour from agriculture to non-agriculture sectors and from
rural to urban areas is a common phenomenon in India. This largely
unskilled labour needs to be absorbed by other sectors of the economy. In
this context, the MSME sub-sector in manufacturing has an important role
to play. The nearly 5.77 crore MSMEs, contributing nearly 40% to India's
manufacturing output, employ around 14 crore people.

Noting the much higher employment potential of MSMEs, the Central


government had launched the ₹20,000 crore Micro Units Development
Refinance Agency (MUDRA) Bank last year. In the wake of the recent trend
of lacklustre global demand and India's weak exports, the MSME sector
needs special attention. Diversification of their exports basket as well as of
export markets is the need of the hour.

67.66. From the below options choose the major causes of jobless growth in India.
1) unskilled people
2) insufficient investment
3) difficulty of doing business
4) rigidity in labour laws

a. 1 and 2 only
b. 1 and 3 only
c. 1, 2 and 3 only
d. All of the above

Answer: d
Explanation:
Jobless growth can have multiple causes. Consider just five of these:
unskilled people, insufficient investment, difficulty of doing business, rigidity
in labour laws (difficulties in firing employees), and inadequate (or
inappropriate) social security systems.
68.67. Agriculture in India is said to have negative employment elasticity. What do
you think are the major reasons for the said phenomenon?
a) Low Agricultural productivity
b) Disguised Employment
c) Both a and b
d) None of the above

Answer key is missing for this question.

Agriculture continues to play a vital role in India's economy. As one of the


most important sectors for providing livelihoods in India, low agricultural
productivity and significant disguised unemployment have hit this labour-
intensive sector hard. Moreover, the vulnerability of the sector to climate
conditions puts this labour at the mercy of the unreliable monsoons.

As a result, there is a general trend of shift in labour out of this sector.


Further, as the Union Budget 2016 identified weakening rural demand as a
dampener to overall economic growth, productivity-enhancing measures
need to be pursued in agriculture sector. This will also be helpful in
arresting the flight of labour away from the sector and add to the
unemployment backlog.

69.68. There is a clear shift in employment from primary to secondary and tertiary
sector which signifies an increase in casual and contract labour as per the recent
economic survey. What are the possible implications of the above trend?
a) Decrease in level of wages
b) Instability of employment
c) Nil/negligible provisions of social security.
d) All of the above
Answer:- d
There is a clear shift in employment to secondary and tertiary sectors from
the primary sector. This reflects increase in both casual labour and
contract workers which has adverse implications on the level of wages,
stability of employment, social security of employees owing to the
‘temporary’ nature of employment.
It also indicates preference by employers away from regular/formal
employment to circumvent labour laws.

70.69. Consider the following statements regarding Labour force participation in


India.
1)Labour force participation rate (LFPR) is defined as the section of
working population in the age group of 16-64 in the economy currently
employed or seeking employment.
2) The All India LFPR of females is much lower than that for males.
Choose the incorrect statements.
a. 1 only
b. 2 only
c. Both 1 and 2
d. None of the above

Answer: D
Explanation:
The Labour Force Participation Rate (LFPR), obtained by dividing the
number of persons in the labour force by total population, is an important
parameter in employment projections and formulation of employment
strategies.

NSSO defines Labour force participation rate (LFPR) as the number of


persons/person days in labour force per thousand persons/person days.

LFPR = No. of persons/person days employed + No. of persons/person


days unemployed x 1000
Total Population
The crucial issue, however, is the basis, or the decision rule, on which a
person is classified as belonging to the labour force. There are four
different concepts used in India in this regard. These are*:

a) Usual Principal Status (UPS)


b) Usual Principal and Subsidiary Status (UPSS)
c) Current Weekly Status (CWS), and
d) Current Daily Status (CDS).
The All India LFPR of females is much lower than that for males.
The North Eastern and Southern States, in general, display high female
LFPR as compared to low levels in Northern States.

71.70. Which of the below causes will lead to structural unemployment in an


economy?
a) Fourth Industrial Revolution
b) Job Outsourcing
c) Both a and b
d) None of the above
Answer:- c
Explanation:-Structural unemployment is caused by forces other than the
business cycle. This means that structural unemployment can last for
decades and may need radical change to redress the situation. If structural
unemployment is not addressed, it can increase the unemployment rate
long after a recession is over and can increase the natural rate of
unemployment.
For example, hundreds of thousands of well-paying manufacturing jobs
have been lost in the United States over the past three decades as
production jobs have migrated to lower-cost areas in China and elsewhere.
This decline in the number of jobs creates a higher natural rate of
unemployment. Growing technology in all areas of life increases future
structural unemployment, since workers without adequate skills will get
marginalized. Even those with skills may face redundancy, given the high
rate of technological obsolescence.
The Fourth Industrial Revolution (4IR) is the fourth major industrial era
since the initial Industrial Revolution of the 18th century. The Fourth
Industrial Revolution is describedas a range of new technologies that are
fusing the physical, digital and biological worlds, and impacting all
disciplines, economies, and industries. It is being associated it with the
"second machine age" in terms of the effects of digitization and AI on the
economy, but added a broader role for advances in biological technologies.
This was discussed in the world economic forum meet.

"Emerging technology breakthroughs" in fields such as artificial intelligence,


robotics, the Internet of Things, autonomous vehicles, 3D printing, quantum
computing and nanotechnology are included in this. The fourth wave of the
industrial revolution is expected to see the heavy implementation of several
emerging technologies with highly potential of disruptive effects.

72.71. Consider the following statements about unemployment.


1) Frictional unemployment isthe unemployment which exists in any
economy due to people being in the process of moving from one job to
another.
2) Full employment implies zero unemployment
Choose the correct statements.
a) 1 only
b) 2 only
c) Both a and b
d) None of the above
Answer:- a
Explanation:-
Full employment is considered to be any acceptable level of unemployment
above 0%. Full employment exists without any cyclical or deficient-demand
unemployment, but does exists with some level of frictional, structural and
voluntary unemployment. Full employment is seen as the ideal employment
rate within an economy and is normally represented by a range of rates
that are specific to regions, time periods and political climates.
A government or economy often defines full employment as any rate
of unemployment below a defined number. If, for example, a country
sets full employment at a 5% unemployment rate, any level of
unemployment below 5% is considered acceptable. Full employment, once
attained, often results in an inflationary period. The inflation is a result of
workers having more disposable income, which would drive prices upward.

Full employment can also be defined as any economic situation that is


devoid of cyclical or deficient-demand unemployment. Cyclical
unemployment is the fluctuating type of unemployment that rises and falls
within the normal course of the business cycle. This unemployment rises
when an economy is in a recession and falls when an economy is growing.
Therefore, for an economy to be at full employment, it cannot be in a
recession that's causing cyclical unemployment.
Deficient-demand unemployment is similar to cyclical unemployment in that
it arises when there isn't enough aggregate demand in an economy to
support full employment. Declining aggregated demand is a characteristic
of a recession. Full employment cannot exist when there isn't enough
demand to support the workforce.

The final three types of unemployment can exist in situations in which full
employment also exists. Structural unemployment arises outside of the
business cycle when there is a skills gap. This unemployment occurs when
there are jobs available but the unemployed population does not have the
knowledge or skill level to perform the required tasks. Technology is a
leading cause of structural unemployment.
Frictional unemployment represents the amount of unemployment that
results from workers who are in between jobs, but are still in the labor
force.
Finally, voluntary unemployment occurs when a person makes a conscious
decision to remain unemployed. This happens when there are jobs
available but a worker cannot find a job of his or her specific choice.

73.72. Which type of unemployment is said to be more prevalent in Indian


Agriculutural sector?
a) Structural unemployment
b) Disguised unemployment
c) Frictional Unemployment
d) Cyclical unemployment
Answer:- b
Agriculture sector, even today, attracts the maximum employment although
its share in the GDP is a meagre 17%. In terms of share in GDP, India is a
service-economy while in terms of employment generation, India is still
stuck with the agrarian-economy status ever since its independence.
Did it ever occur to you that how can agriculture, despite its little
contribution to GDP, attracts the maximum employment? Well if this
question ponders your head often, let me clear all doubts.
The agriculture sector experiences a massive unemployment in two major
forms, primarily:
1. Seasonal Unemployment

2. Disguised unemployment

Now, let’s discuss these concepts one by one to create clarity.


Seasonal unemployment occurs primarily in the agriculture sector and
there are very instances of experiencing such forms of unemployment in
the urban sector or in other sectors of the economy. It is a kind of
unemployment when people are employed only for a few months or a
particular season of the year and the individual remains unemployed during
the remaining months of the year. Farmers often employ additional workers
during the sowing season or the harvesting season and in the remaining
time period that person is unemployed.
The next is disguised unemployment. This is a very interesting type of
unemployment because as the name suggests, this unemployment is
hidden or not seen easily. It is that form of unemployment wherein more
people are employed as compared to the requirement level. Understand
this with an example.
Suppose a farmer employs 5 people on his field whereas only 3 people
were sufficient for that land. Hence, the additional 2 workers are not adding
to the production process and so, these 2 additional workers would be
considered disgusedly unemployed.
This is a very basic idea of the concept of disguised unemployment. Such
forms may also be prevalent in urban sectors wherein more labour is
employed say, in a factory, as compared to the number of people
employed. Hence the marginal productivity of each additional labour falls
and eventually becomes negative which no firm tends to be at.

74.73. Identify the correctly matched pairs of schemes and their corresponding
nodal ministries.
1) Swachh Bharat Mission Urban:Ministry of Urban Development
2) Swachh Bharat Mission Gramin: Ministry of Drinking Water and
Sanitation

3) SwachhVidayalaya :Minsitry of Human Resource and Development


4)Swachh Bharat Kosh :Minsitry of Finance
a) 1 and 2 only
b) 1, 2 and 3 only
c) 1 and 4 only
d) All of the above
Answer:- d
Explanation:-
The urban component of the Swachh Bharat mission will be implemented
by the Ministry of Urban Development, and the rural component by the
Ministry of Drinking Water and Sanitation.
The programme includes elimination of open defecation, conversion of
unsanitary toilets to pour flush toilets, eradication of manual scavenging,
municipal solid waste management and bringing about a behavioural
change in people regarding healthy sanitation practices.
The Nirmal Bharat Abhiyan has been restructured into the Swachh Bharat
Mission (Gramin). The mission aims to make India an open defecation free
country in Five Years. It seeks to improve the levels of cleanliness in rural
areas through Solid and Liquid Waste Management activities and making
Gram Panchayats Open Defecation Free (ODF), clean and
sanitised.Technology will be used on a large scale to convert waste into
wealth in rural India in the forms of bio-fertilizer and different forms of
energy. The mission is to be executed on war footing with the involvement
of every gram panchayat, panchayat samiti and ZilaParishad in the country,
besides roping in large sections of rural population and school teachers
and students in this endeavor.
The Ministry of Human Resource Development has launched
SwachhVidyalayaProgramme under Swachh Bharat Mission with an
objective to provide separate toilets for boys and girls in all government
schools within one year. The programme aims at ensuring that every
school in the country must have a set of essential interventions that relate
to both technical and human development aspects of a good Water,
Sanitation and Hygiene Programme.
The Swachh Bharat Kosh (SBK) has been set up to facilitate and
channelize individual philanthropic contributions and Corporate Social
Responsibility (CSR) funds to achieve the objective of Clean India (Swachh
Bharat) by the year 2019.
Contributing to the Swachh Bharat Kosh that has been set up under the
aegis of Ministry of Finance to receive financial contribution which is 100%
tax exempted.
The Kosh will be used to achieve the objective of improving cleanliness
levels in rural and urban areas, including in schools. The allocation from the
Kosh will be used to supplement and complement departmental resources
for such activities.

75.74. Which of the following is a ranking exercise based on cleanliness status of


Rural and urban areas?
a) SwachhSurvekshan
b) Swachh Iconic places
c) SwachhSwasthSarvatra
d) Both a and c
Answer:- a
Explanation:-
SwachhSurvekshan is a ranking exercise taken up by the Government of
India to assess rural and urban areas for their levels of cleanliness and
active implementation of Swachhata mission initiatives in a timely and
innovative manner.
o The objective of the survey is to encourage large scale citizen participation
and create awareness amongst all sections of society about the importance
of working together towards making towns and cities a better place to live
in. Additionally, the survey also intends to foster a spirit of healthy
competition among towns and cities to improve their service delivery to
citizens, towards creating cleaner cities and towns.

o The Ministry of Urban Development, Government of India takes up the


SwachhSurvekshan in urban areas and the Ministry of Drinking Water and
Sanitation in rural areas. The Quality Council of India (QCI) has been
commissioned the responsibility of carrying out the assessment.

SwachhSwasthSarvatrais a joint initiative of the Ministry of Health and


Family Welfare and the Ministry of Drinking Water and Sanitation to
achieve better health outcomes through improved sanitation and increased
awareness and healthy lifestyles.
The three key components of SwachhSwasthSarvatra are:
 Community Health Centres (CHCs) in ODF blocks supported to achieve Kayakalp
certification
 Gram Panchayat of Kayakalp Primary Health Centres (PHCs) prioritized to
become ODF
 Training in WASH (Water, Sanitation and Hygiene ) of CHC/PHC nominees

SwasthyaRakshaProgrammelaunched by the AYUSH Ministry to promote


health and health education in villages
The Swachh Iconic Places is an initiative under the Swachh Bharat
Mission. It is a special clean-up initiative focused on select iconic heritage,
spiritual and cultural places in the country. The initiative is being
coordinated by the Ministry of Drinking Water and Sanitation in association
with the Ministry of Urban Development, Ministry of Culture, Ministry of
Tourism and the concerned State governments.

76.75. Consider the following statements about SwachhtaUdyamiYojana.


1) This Scheme has twin objective of cleanliness and providing livelihood to
SafaiKaramcharis and liberated Manual Scavengers to achieve the overall
goal of “Swachh Bharat Abhiyan”.
2)The Ministry of Social Justice and Empowerment launched the
SwachhtaUdyamiYojana.
Choose the correct statements.
a) 1 only
b) 2 only
c) Both a and b
d) None of the above
Answer:- c
Explanation:-
The SwachhtaUdyamiYojana extends financial assistance for Construction,
Operation and Maintenance of Pay and Use Community Toilets in Public
Private Partnership (PPP) Mode and Procurement and Operation of
Sanitation related Vehicles. The Ministry of Social Justice and
Empowerment launched the SwachhtaUdyamiYojana (SUY) on
02nd October 2014.
Objectives
This Scheme has twin objective of cleanliness and providing livelihood to
SafaiKaramcharis and liberated Manual Scavengers to achieve the overall
goal of “Swachh Bharat Abhiyan”
Construction, Operation and Maintenance of Pay and Use Community
Toilets
 Provision of easy accessibility of the community latrines to the households
(not having any such facilities in their houses) and for floating population in
public places with high footfall e.g. bus stands, railway stations, markets
etc.

 To ensure proper maintenance of the facilities, so created, by the


entrepreneurs, who would have stake in this venture.

 To prevent the necessity of manual scavenging.

Procurement & Operation of Sanitation related Vehicles


 To create appropriate infrastructure for tapping the underutilized potential.
 To create facilities for collection of garbage from the source.

 To create employment opportunities for the target group of


safaikaramcharis /manual scavengers.

77.76. Consider the following statements about WASH Scheme.


1) ‘WASH’ in school aims to make a visible impact on the health and
hygiene of children through improvement in their health and hygiene
practices.
2) A key feature of the‘Clean India: Clean Schools’campaign is to ensure
that every school in India has a set of functioning and well maintained
water, sanitation and hygiene facilities.
Choose the correct statements.
a) 1 only
b) 2 only
c) Both a and b
d) None of the above

Answer:- c
Explanation:-
Swachh Bharat: SwachhVidyalaya is the national campaign driving
‘Clean India: Clean Schools’. A key feature of the campaign is to ensure
that every school in India has a set of functioning and well maintained
water, sanitation and hygiene facilities. Water, sanitation and hygiene in
schools refers to a combination of technical and human development
components that are necessary to produce a healthy school environment
and to develop or support appropriate health and hygiene behaviours. The
technical components include drinking water, hand washing, toilet and soap
facilities in the school compound for use by children and teachers. The
human development components are the activities that promote conditions
within the school and the practices of children that help to prevent water,
hygiene and sanitation related diseases.
School sanitation and hygiene depend on a process of capacity
enhancement of teachers, community members, Non-Governmental
Organisations (NGOs) and Community Based Organisations (CBOs)
and education administrators. Water, sanitation and hygiene in school
aims to make a visible impact on the health and hygiene of children through
improvement in their health and hygiene practices, and those of their
families and the communities. It also aims to improve the curriculum and
teaching methods while promoting hygiene practices and community
ownership of water and sanitation facilities within schools. it improves
children’s health, school enrolment, attendance and retention and paves
the way for new generation of healthy children. It is the role of
policymakers, government representatives, citizens and parents to make
sure that every child attends a school that has access to safe drinking
water, proper sanitation and hygiene facilities. This is every child’s right.
SwachhVidyalaya – The Essential Elements
Every school in the country must have a set of essential interventions that
relate to both technical and human development aspects of a good Water,
Sanitation and Hygiene Programme.
78.77. The government has adopted a single day strategy called NATIONAL
DEWORMING DAY (NDD) and it is related to which of the following?
a) Soil Transmitted helminthes
b) Pest Management
c) Biomanure
d) None of the following
Answer:- a

Explanation:-
World Health Organization estimates that 241 million children between the
ages of 1 and 14 are at risk of parasitic intestinal worms in India, known as
Soil-Transmitted Helminths (STH).
 There are three main types of STH that infect people: roundworm
(Ascarislumbricoides), whipworm (Trichuristrichiura) and hookworms
(Necatoramericanus and Ancylostomaduodenale).

Harms associated with having worms


Effect of STH on the nutritional status of children
Worms impair the nutritional status of people they infect in multiple ways:
 Worms feed on host tissues, including blood, which leads to a loss of iron
and protein and often contributes to anaemia.

 Worms can increase the malabsorption of nutrients; roundworm may


compete for Vitamin A in the intestine.

 Some worms can cause a loss of appetite, reducing nutritional intake and
physical fitness.

 Some worms can cause diarrhea and dysentery.

Development and educational consequences of worms in children


Worms have negative effects on the mental and physical development of
children. Children with worms are often underweight and have stunted
growth. Heavy infections often make children too sick or too tired to
concentrate at or even attend school. Long term, children not treated for
worms are shown to earn less as adults.
Effect of worms on child mortality
Intestinal worm infections affect child morbidity, not mortality. There is not
rigorous evidence that suggests that worms affect child mortality but there
is ample evidence that worms fundamentally affect the quality of children’s
lives and negatively impact their access to health, education and
livelihoods.
With an aim to intensify efforts towards STH control among children in
India, the Ministry of Health & Family Welfare, Government of India (GoI)
observes the National Deworming Day (NDD) on 10th February every year.
To combat STH infections, government has adopted a single day strategy
called NATIONAL DEWORMING DAY (NDD). The programme is being
implemented through the combined efforts of Department of School
Education and Literacy under Ministry of Human Resource and
Development, Ministry of Women and Child Development and Ministry of
Drinking Water and Sanitation.
During NDD single dose of Albendazole is administered to children from
under 1 to 19 years of age group through the platforms of schools and
aanganwadicenters. Albendazole is an extremely safe drug that has been
used for decades by millions of people around the world and in India, with
no or minimal side effects. Side effects such as nausea and vomiting can
manifest in the children with high worm load.
National deworming DAY is a commendable step that will go a long way in
ensuring healthy India. On India’s glorious record sheet of eradicating
Polio, Guinea worm, Small pox, Maternal and neonatal tetanus, this may
become yet another entry.

79.78. Which of the following provisions is/are part of the MGNREG Act?
1)Right to get unemployment allowance
2)Social Audit of MGNREGA works is mandatory
3) It provides a legal guarantee for wage employment
a. 1 and 2 only
b. 1 and 3 only
c. 1, 2 and 3 only
d. 2 and 3 only
Answer: c
Explanation:
The Mahatma Gandhi National Rural Employment Guarantee Act
(MGNREGA), also known as Mahatma Gandhi National Rural Employment
Guarantee Scheme (MNREGS) is Indian legislation enacted on August 25,
2005. The MGNREGA provides a legal guarantee for one hundred days of
employment in every financial year to adult members of any rural
household willing to do public work-related unskilled manual work at the
statutory minimum wage. The Ministry of Rural Development (MRD), Govt
of India is monitoring the entire implementation of this scheme in
association with state governments
Key facts that users should know about MNREGA
1. MGNREGA guarantees hundred days of wage employment in a financial
year, to a rural household whose adult members volunteer to do unskilled
manual work.

2. Individual beneficiary oriented works can be taken up on the cards of


Scheduled Castes and Scheduled Tribes, small or marginal farmers or
beneficiaries of land reforms or beneficiaries under the Indira AwaasYojana
of the Government of India.

3. Within 15 days of submitting the application or from the day work is


demanded, wage employment will be provided to the applicant.

4. Right to get unemployment allowance in case employment is not provided


within fifteen days of submitting the application or from the date when work
is sought.

5. Receipt of wages within fifteen days of work done.

6. Variety of permissible works which can be taken up by the Gram


Panchayaths.

7. MGNREGA focuses on the economic and social empowerment of women.

8. MGNREGA provides “Green” and “Decent” work.

9. Social Audit of MGNREGA works is mandatory, which lends to


accountability and transparency.
10. MGNREGA works address the climate change vulnerability and
protect the farmers from such risks and conserve natural resources.

11. The Gram Sabha is the principal forum for wage seekers to raise their
voices and make demands. It is the Gram Sabha and the Gram Panchayat
which approves the shelf of works under MGNREGA and fix their priority.

80.79. Which of the following is/are GIS (Geographical Information System) used
to enable e-governance in MGNREGS?
a) NREGAsoft
b) GeoMGNREGA
c) Both a and b
d) None of the above
Answer:- b
Explanation:-
 NREGAsoft is an MIS Application which envisions implementing e-
Governance across State, District and three tiers of Panchayati Raj
Institutions.

 It empowers the common man using the information technology as a


facilitator.

 NREGAsoft provides information to citizen in compliance with the right to


information Act (RTI Act). It makes available all the documents like Muster
Rolls, registration application register, job card/employment register/muster
roll issue register, muster roll receipt register which are hidden from public
otherwise.

 Facilitate faster information exchange between the various stakeholders


through the network.

E-Governance for Masses:


 Computerisation of all NREGA activities

 All information available for public access

 Accuracy ensured in payment of wages to workers


 Lodging of complaints through Grievance Redressal System

 Facilitation of Social Audit

GeoMGNREGA is a software solution to Photo Geotag all completed


assets under Mahatma Gandhi by the use of space technology and to
integrate the details of the wage and material into the public domain to
ensure transparency and accountability.
The objective of GeoMGNREGA is to create a Geographical Information
System (GIS) solution to visualize, analyze and explore the data of assets
created under the MGNREGA. It essentially enables to view the assets
created under MGNREGA across India on a map. GeoMGNREGA
leverages ‘Bhuvan’, the software platform developed by National Remote
Sensing Centre (NRSC) of ISRO for this purpose. While the data of assets
is taken from the NREGASoft application of Ministry of Rural Development.
The unique capabilities of NREGASoft and Bhuvan are integrated into one
Geospatial Asset Management & Planning tool and is named as
‘GeoMGNREGA’.
BhuvanGeoMGNREGA portal:
It is a web-based application mgnrega allowing users to visualize the geo-
tagged locations. It also allows authorized users to moderate the collected
geo-tagged assets before it is made visible to the public and get the
summary of the assets geo-tagged in their respective jurisdiction.

81.80. Which of the following Finance and development corporations fall under
the Ministry of Social Justice and Empowerment?
1)National Backward Classes Finance and Development Corporation
2)National SafaiKaramcharis Finance & Development Corporation
3)National Scheduled Castes Finance and Development Corporation
a) 2 only
b) 1, 2 and 3 only
c) 2 and 3 only
d) None of the above

Answer: - b
Explanation:-
National Scheduled Castes Finance and Development Corporation

 VISION : "To be the leading catalyst in systematic reduction of poverty


through socio-economic development of Scheduled Castes living below
double the poverty line, working in an efficient, responsive and
collaborative manner with channelizing agencies and other development
partners."

 MANDATE : Providing concessional finance for setting up of self-


employment projects and skill-training grants to unemployed SC persons
living below Double the Poverty Line.

 NSFDC : NSFDC is an institution under Ministry of Social Justice &


Empowerment, Government of India for financing, facilitating and
mobilizing funds for the economic empowerment of persons belonging to
the Scheduled Castes families living below Double the Poverty Line.
NSFDC finances income generation schemes for the target group through
the State Channelising Agencies (SCAs) nominated by respective
State/UT Governments.

 BROAD OBJECTIVE : NSFDC is the apex institution for financing,


facilitating and mobilizing funds from other sources and promoting the
economic development activities of the persons belonging to the
Scheduled Castes living below double the poverty line.

National SafaiKaramcharis Finance & Development


Corporation(NSKFDC), a wholly owned Govt. of India Undertaking under
the Ministry of Social Justice & Empowerment (M/o SJ&E) was set up
on 24th January 1997 as a Company “Not for Profit” under Section 25 of
the Companies Act, 1956. NSKFDC is in operation since October, 1997,
as an Apex Corporation for the all round socio-economic upliftment of the
SafaiKaramcharis, Scavengers and their dependants throughout
India,through various loan and non-loan based schemes.
Apart from operating various loan and non-loan based schemes for the
upliftment of the target group, NSKFDC is playing a vital role in elimination
of manual scavenging - the worst surviving symbol of
untouchability. NSKFDC has been designated as the Nodal Agency for
implementation of the Central Sector Self Employment Scheme for
Rehabilitation of Manual Scavengers (SRMS) under the aegis of the
Ministry of Social Justice & Empowerment.
National Backward Classes Finance & Development Corporation
(NBCFDC) is a Govt. of India Undertaking under the aegis of Ministry of
Social Justice and Empowerment. NBCFDC was incorporated under
Section 25 of the Companies Act 1956 on 13th January 1992 as a
Company not for profit with an objective to promote economic and
developmental activities for the benefit of Backward Classes and to assist
the poorer section of these classes in skill development and self
employment ventures. NBCFDC provides financial assistance through
State Channelising Agencies (SCAs) nominated by the State Governments/
UTs. NBCFDC also provides Micro Financing through SCAs/ Self Help
Groups (SHGs). The Corporation can assist a wide range of income
generating activities to assist the poorer section of these classes in skill
development and self-employment ventures under following broad sectors :
1. Agriculture and Allied Activities
2. Small Business / Artisan and Traditional Occupation
3. Service / Transport Sector etc.
4. Technical and Professional Trades/Courses

82.81. Consider the following statements about the Pradhan MantriAdarsh Gram
Yojana (PMAGY)
1) It aims to ensure integrated development of the selected villages into
“model villages”.
2) It is being implemented in SC majority villages to eliminate disparity
between SC and non-SC population in terms of common socio- economic
indicators (e.g. literacy rate, completion rate of elementary education,
IMR/MMR, ownership of productive assets, etc.).
Choose the correct statements.
a) 1 only
b) 2 only
c) Both a and b
d) None of the above
Answer: - c
Explanation:-
Pradhan MantriAdarsh Gram Yojana (PMAGY): •The pilot phase of the
Scheme was launched in 2009-10 for the integrated development of 1000
SC majority villages across 5 States in the country. •The Scheme was
expanded in January 2015 to cover 1500 new SC majority villages across
10 States of the country
Pradhan MantriAdarsh Gram Yojana (PMAGY), a Government of India
initiative for the empowerment of deprived sections, aims to achieve
integrated development of selected villages through convergent
implementation of all relevant Central and State schemes. The scheme
was launched in March, 2010 on a pilot basis for the integrated
development of 1000 villages each with more than 50% SC population.
Objective
To ensure integrated development of the selected villages into “model
villages” so that, inter alia,
1. They have all requisite physical and social infrastructure for their socio-
economic development, and satisfy the norms mentioned in the vision of an
adarsh gram to the maximum possible extent.

2. Disparity between SC and non-SC population in terms of common socio-


economic indicators (e.g. literacy rate, completion rate of elementary
education, IMR/MMR, ownership of productive assets, etc.) is eliminated,
the indicators are raised to at least the level of the national average, and

1. All BPL families, especially those belonging to SCs, have food and
livelihood security, and are enabled to cross the poverty line and earn an
adequate livelihood

2. All children complete at least eight years of education, and


3. Incidence of malnutrition, especially among children and women, is
eliminated.

4. Untouchability, discrimination, segregation, and atrocities against SCs are


eliminated, as are other social evils like discrimination against
girls/women,alcoholism and substance (drugs) abuse, etc., and all sections
of society are able to live with dignity and equality, and in harmony with
others.

Vision of an “Adarsh Gram” (Model Village)


A Model village is one which has adequate physical and institutional
infrastructure, in which minimum needs of all sections of the society are
fully met; they live in harmony with each other, as also with the
environment, and a village which is progressive and dynamic. These
villages should be covered with all the facilities necessary for dignified
living, creating thereby an environment in which all its residents are
enabled to utilise their potential to the fullest.

83.82. Which of the following is/are the key features of the Scheduled Castes and
the Scheduled Tribes (Prevention of Atrocities) Amendment Act, 2015?
1) It penalizes willful neglect by public servants.
2)Establishment of Exclusive Special Courts and specification of Exclusive
Special Public Prosecutors also, to exclusively try the offences.
3)Rights of victims and witnesses

a) 2 only
b) 1, 2 and 3 only
c) 2 and 3 only
d) None of the above
Answer:- b
Explanation:-
The Scheduled Castes and the Scheduled Tribes (Prevention of Atrocities)
Amendment Act, 2015 seeks to ensure more stringent provisions for
prevention of Atrocities against Scheduled Castes and the Scheduled
Tribes. The Act is an amendment to the Principal Act, namely, the
Scheduled Castes and the Scheduled Tribes (Prevention of Atrocities)
(PoA) Act, 1989 and is being enforced with effect from January 26, 2016.
Key features
The key features of the Scheduled Castes and the Scheduled Tribes
(Prevention of Atrocities) Amendment Act, 2015, are:

Actions to be treated as offences - The Act outlines actions (by non SCs
and STs) against SCs or STs to be treated as offences. The Amendment
Act amends certain existing categories and adds new categories of actions
to be treated as offences. New offences added under the Act include: (a)
garlanding with footwear, (b) compelling to dispose or carry human or
animal carcasses, or do manual scavenging, (c) abusing SCs or STs by
caste name in public, (d) attempting to promote feelings of ill-will against
SCs or STs or disrespecting any deceased person held in high esteem,
and (e) imposing or threatening a social or economic boycott.
Assaulting or sexual exploiting an SC or ST woman is an offence under
the Act - The Amendment Act adds that: (a) intentionally touching an SC or
ST woman in a sexual manner without her consent, or (b) using words, acts
or gestures of a sexual nature, or (c) dedicating an SC or ST women as a
devadasi to a temple, or any similar practice will also be considered an
offence. Consent is defined as a voluntary agreement through verbal or
non-verbal communication.
Preventing SCs or STs from undertaking the following activities will be
considered an offence - (a) using common property resources, (c) entering
any place of worship that is open to the public, and (d) entering an
education or health institution.
Addition of presumption to the offences – The court shall presume that
the accused was aware of the caste or tribal identity of the victim if the
accused had personal knowledge of the victim or his family, unless the
contrary is proved.
Role of public servants - The Act specifies that a non SC or ST public
servant who neglects his/her duties relating to SCs or STs shall be
punishable with imprisonment for a term of six months to one year. The
Amendment Act specifies these duties, including: (a) registering a
complaint or FIR, (b) reading out information given orally, before taking the
signature of the informant and giving a copy of this information to the
informant, etc.
Addition of certain IPC offences like hurt, grievous hurt, intimidation,
kidnapping etc., attracting less than ten years of imprisonment, committed
against members of Scheduled Caste/Scheduled Tribe, as offences
punishable under the PoA Act. Presently, only those offences listed in IPC
as attracting punishment of 10 years or more and committed on members
of Scheduled Caste/Scheduled Tribe are accepted as offences falling
under the PoA Act.
Establishment of Exclusive Special Courts and specification of
Exclusive Special Public Prosecutors also, to exclusively try the offences
under the PoA Act to enable speedy and expeditious disposal of cases.
Rights of victims and witnesses- The Amendment Act adds a chapter
on the rights of victims and witness. It shall be the duty of the state to make
arrangements for the protection of victims, their dependants and witnesses.
The state government shall specify a scheme to ensure the implementation
of rights of victims and witnesses.
The courts established under the Act may take measures such as: (a)
concealing the names of witnesses, (b) taking immediate action in respect
of any complaint relating to harassment of a victim, informant or witness,
etc. Any such complaint shall be tried separately from the main case and
be concluded within two months.

84.83. Which of the following is a scheme to provide financial assistance to


safaikaramcharisin the form of loan for the activities which could tackle the
climate change alongwith income generation?
a) Sanitary mart scheme
b) Green Business Scheme
c) SwachataUdyamiYojana
d) MahilaAdhikaritaYogana

Answer:-b
Explanation:-
Green Business Scheme
Climate change, which has become one of the greatest challenges for the
world, is the consequence of unchecked pollution. With a view to mitigate
the impact of climate change on human beings, it is necessary to bring
down pollution level, for which it is necessary to promote such business
activities which can tackle the climate change and also help in income
generating.
Objective: To provide financial assistance in the form of loan for the
activities which could tackle the climate change alongwith income
generation. The income generating activities which could mitigate the
Green House effect or could be classified under adaptation initiatives,
would be covered under the scheme.
Eligibility: SafaiKaramcharis, Scavengers and their dependants
Indicative schemes : Battery electric vehicle (E-rickshaw), Compressed
air vehicle, Solar energy gadgets, Poly houses
MahilaAdhikaritaYojana (MAY)
 Under the MahilaAdhikaritaYojana (MAY) loan is provided through State
Channelizing Agencies (SCAs), Regional Rural Banks (RRBs)
and Nationalized Bank to the target group.

 Loan under this schemes is provided to SafaiKaramchari and Scavenger


women and their dependant daughters

The “SwachhtaUdyamiYojana” is for extending financial assistance for


Construction, Operation & Maintenance of Pay and Use Community Toilets
in Public Private Partnership (PPP) Mode and Procurement & Operation of
Sanitation related Vehicles .
Sanitary Marts Scheme
A Sanitary Mart is a one-stop-shop for all things for sanitation and
hygiene. It is a shopping place where the sanitary needs of the common
man could be met. It serves both as a shop and as a service centre.

85.84. Which of the following are the major traits of depression phase?
a. Low aggregate demand
b. High inflation
c. Increasing unemployment rate
d. Forced labour cuts or retrenchment by industries
Choose the correct options:
i. 1 , 2 and 4 only
ii. 1, 2 and 3 only
iii. 1 ,3 and 4 only
iv. 1,2,3 and 4
Solution: C
Traits of depression:
1) An extremely Low aggregate demand in the economy
2) Comparatively low inflation
3) Increasing unemployment rate
4) Forced labour cuts or retrenchment by industries( to cut down production or be
competitive in the market)

86.85. Which of the following are the major traits of recovery phase?
a. An Upturn in aggregate demand
b. Increase in level of production
c. Inflation moves downward
d. Decreasing unemployment rate
Choose the correct options:
i. 1, 2 and 4 only
ii. 1, 2 and 3 only
iii. 1,3 and 4 only
iv. 1,2 ,3 and 4
Solution: A
1) An Upturn in aggregate demand which has to be accompanied by Increase in the level
of production
2) Increase in level of production
3) Inflation and demand for goods and services goes upward
4) Decreasing unemployment rate

87.86. Consider the following statements regarding boom period of business cycle:
a. A strong upward fluctuations in the economic activities is called boom
b. There will be an accelerated and prolonged increase in demand
c. Inflation start going upward
Choose the correct options:
i. 1 and 2 only
ii. 1 and 3 only
iii. 1,2 and 3
iv. 2 and the only
Soultion: C
1)A strong upward fluctuations in the economic activities is called boom
2)There will be an accelerated and prolonged increase in demand
3)Inflation start going upward
4) Economy heats up and demand supply gap is visible

88.87. Black money includes:


a. Wealth earned through illegal means
b. Legal income that is concealed from public authorities
Choose the correct options:
i. 1 Only
ii. 2 Only
iii. Both 1 and 2
iv. Neither 1 nor 2
Solution: C
Black Money includes:
 Wealth earned through illegal means
 Legal income that is concealed from public authorities: to evade payment of taxes
(income tax, excise duty, sales tax, stamp duty, etc); o evade payment of other
statutory contributions; to evade compliance with other laws and administrative
procedures.

Underlying factors for generation of Black Money

 ‘Criminal’ component: This include proceeds from a range of activities including


racketeering, trafficking in counterfeit and contraband goods, smuggling, production
and trade of narcotics, forgery, illegal mining, illegal felling of forests, illicit liquor
trade, robbery, kidnapping, human trafficking, sexual exploitation and prostitution,
cheating and financial fraud, embezzlement, drug money, bank frauds, and illegal
trade in arms.

 Failure of reporting: Significant amount of black money, however, is generated


through legally permissible economic activities, which are not accounted for and
disclosed or reported to the public authorities as per the law or regulations, thereby
converting such income into black money.

 Black Money by manipulation of accounts:

 Out of book transaction to evade taxes


89.88. Which of the following are the possible effects of black money in the economy?
1) It affects the financial system of the country.
2)It will affect the credibility of our country.
3) The money generated in the form of black money is often routed for terrorism
4) huge loss of money for government exchequer because of tax loss for the country
5) Real estate prices may get reduced.
Choose the correct options:
A) 1 ,2 ,4 and 5 only
B) 1,2 , 3and 4 only
C) 2,3 4 and 5 only
D) 1,2,3,4 and 5 only
Ans: B
Effects of Black Money

The after effects of black money in our economy is manifold and have serious
consequences. Some of the negative impacts are listed below.

 It affects the financial system of the country. RBI may not be able to control the money
supply in the economy and thereby leading to higher inflation in the system. The
persistent level of high inflation is detrimental to the economy as it will erode the value
of money and eats up investments and savings.
 It will affect the credibility of our country.
 The money generated in the form of black money is often routed for terrorism and
hence, it becomes an issue of national security.
 A parallel economy will be created in the country and hence, governance will be
ineffective to a large extent. We have the examples like Mexico where a parallel
economy is rampant due to illegal drug trafficking.
 There is a huge loss of money for government exchequer because of tax loss for the
country
 Real estate prices may shoot up. This may result in asset bubble which is detrimental to
an economy.
Day 2
90.89. Consider the following statements regarding Pradhan
MantriGaribKalyanYojana, 2016 (PMGKY)
1)The scheme provides an opportunity to declare unaccounted wealth
and black money in a confidential manner and avoid prosecution.
2)A fine of 50% is levied on the undisclosed income.
3)Undisclosed income can be in the form of cash, jewellery, stock,
immovable property, or deposits in overseas accounts.
Which of the above statements are true?
OPTIONS
a)1 and 2
b)2 and 3
c)1 and 3
d)All the above
SOLUTION:a
Pradhan MantriGaribKalyanYojana, 2016 (PMGKY)( Prime Minister's
Poor welfare scheme) is an amnesty scheme launched by the Narendra
Modi led Government of India in December 2016 on the lines of the Income
declaration scheme, 2016 (IDS) launched earlier in the year. A part of
the Taxation Laws (Second Amendment) Act, 2016, the scheme provides
an opportunity to declare unaccounted wealth and black money in a
confidential manner and avoid prosecution after paying a fine of 50% on
the undisclosed income. An additional 25% of the undisclosed income is
invested in the scheme which can be refunded after four years, without any
interest.
Valid from December 16, 2016 to March 31, 2017, the scheme was availed
to declare income in the form of cash or bank deposits in Indian bank
accounts and not in the form of jewellery, stock, immovable property,
or deposits in overseas accounts.

Not declaring undisclosed income under the PMGKY will attract a fine of
77.25% if the income is shown in tax returns. In case the income is not
shown in tax returns, it will attract a further 10% penalty followed by
prosecution.

91.90. With which of the following countries had India signed Double taxation
Avoidance agreement?
1)Mauritius
2)Singapore
3)US
4)UK
5)SaudiArabia
6)Australia
OPTIONS
a)1 and 2
b)1 2 3 and 4
c)3 and 4
d)All the above

SOLUTION:d
India has DTAA with over 80 countries; it plans to sign such treaties with
more countries. The major countries with which it has signed the DTAA are
the US, the United Kingdom, theUAE, Canada, Australia, Saudi
Arabia, Singapore and New Zealand.

92.91. Recently EU released 17 tax haven blacklist countries. Which of the


following countries are tax havens?
1)South Korea
2)United Arab Emirates
3)Mongolia
4) Panama
5)Marshall Island
OPTIONS:
a)4 and 5
b)1, 4 and 5
c) 1,3 and 4
d)All the above

Solution:d
The EU has named 17 countries in its first ever tax haven blacklist and put
a further 47 on notice
The blacklist includes South Korea, Mongolia, Namibia, Panama, Trinidad
& Tobago, Bahrain and the United Arab Emirates,Guam(the US territory in
the Pacific)American Samoa, Barbados, Grenada, Macau, the Marshall
Islands, Palau, St Lucia, Samoa and Tunisia.
93.92. Consider the following statements regarding PANAMA and PARADISE
papers
1)Both the papers revealed offshore activities of some of the world's most
powerful people and companies.
2) International Consortium of Investigative Journalists (ICIJ) were involved
in unearthing and publicizing both the papers.
Which of the statements are not true?
OPTIONS
a) 1 only
b) 2 only
c) Both 1 and 2
d)None of the above
SOLUTION:d
1)Both the papers revealed offshore activities of some of the world's most
powerful people and companies.
The Panama Papers are 11.5 million leaked documents that detail
financial and attorney–client information for more than 214,488 offshore
entities.
The Paradise Papers are a set of 13.4 million confidential electronic
documents relating to offshore investments.
2) International Consortium of Investigative Journalists (ICIJ) were
involved in unearthing and publicizing both the papers.

94.93. Which is the final stage of money laundering after which the money
appears to be legally earned?
a)Placement
b)Layering
c)Integration
d)Assimilation
SOLUTION:C
Integration
This is the stage where the funds are returned to the legitimate economy
for later extraction. Examples include investing in a company, purchasing
real estate, luxury goods, etc.

This is the final stage in the process. The launderer makes it appear to
have been legally earned and accomplishes integration of the “cleaned”
money into the economy

DAY 3
95.94. Arrange the stages/Phases of business cycle in order
1)Recession
2)Recovery
3)Depression
4)Boom
OPTIONS
a)1 2 3 4
b)1 3 2 4
c)3 2 4 1
d)1 2 4 3
SOLUTION:C
96.95. What is double-dip recession?

a)recession followed by a short-lived recovery, followed by


another recession
b)Recession in both national and International Business cycle
c)Recession dipping to extremely low levels that recovery takes
many years
d)None of the above
SOLUTION:a
A double-dip recession refers to a recession followed by a short-
lived recovery, followed by another recession.

97.96. Consider the following statements regarding shell companies


1) A shell corporation is a corporation without active business
operations or significant assets.
2)They are multiple layers of companies that have been created
for the purpose of diverting money or for money laundering.
3)Shell companies are necessarily illegal entities.
OPTIONS:
a)1and 2 only
b)2 and 3 only
c)1 and 3 only
d)All the above
SOLUTION:a
The Securities and Exchange Board of India (Sebi) directed stock
exchanges to initiate action against 331 suspected shell
companies and bar them from trading. Further, MCA cancelled
the registration of around 2,09,032 defaulting companies and the
Ministry of Finance directed banks to restrict operations of bank
accounts of such companies by the directors of such companies
or their authorized representatives.
A shell corporation is a corporation without active business
operations or significant assets.
These types of corporations are not all necessarily illegal.
Legitimate reasons for a shell corporation include such things as
a startup using the business entity as a vehicle to raise, funds,
conduct a hostile takeover or to go public.
But they are sometimes used illegitimately, such as to disguise
business ownership from law enforcement or the public .
Shell companies include multiple layers of companies that have
been created for the purpose of diverting money or for money
laundering.

98.97. What steps the government may take to rescue the economy
from the phase of recession
1)Increase the direct and indirect taxes so that government will
have more money to spent to act as a stimulus in the economy.
2)Follow dear money policy
OPTIONS:
a)1 only
b)2 only
c)Both 1 and 2
d)None

Solution:d
A)Decrease the direct and indirect taxes so that consumers have
high disposable incomes on one hand and goods become
cheaper on the other hand, thus there is a hope that the demand
might pick up.
B)Follow cheap money policy by slashing down interest rates and
lending procedure is also liberalized, so the economy picks up.

99.98. Consider the following statements regarding ‘General Anti-


Avoidance Rules’ in India
1)It is to curb ill intended tax avoidance.
2)Helps tax officials to target participatory notes.
OPTIONS:
a)1 only
b)2 only
c)Both 1 and 2
d)None

Solution:C
The General Anti Avoidance rules allows tax officials to deny tax
benefits, if a deal is found without any commercial purpose other
than tax avoidance.
It allows tax officials to target participatory notes. Under GAAR,
the investor has to prove that the participatory note was not set to
avoid taxes.
It also allows officials to deny double taxation avoidance benefits,
if deals made in taxhavens like Mauritius were found to be
avoiding taxes.

DAY 4
100.99. Consider the following statements regarding Financial Intelligence
Unit
1)It is a wing/Unit under Intelligence Bureau
2)It functions under the aegis of Ministry of Home Affairs
3)It reports directly to Economic Intelligence Council(EIC) headed
by Home Minister
Which of the following statements are not correct?
OPTIONS
a)1 and 2
b)2 and 3
c)1 only
d)All the above
SOLUTION:d
Financial Intelligence Unit functions under the aegis of Ministry of
Finance
FIU-IND is an independent body reporting directly to the
Economic Intelligence Council (EIC) headed by the Finance
Minister.

You might also like